You are on page 1of 34

VISIONIAS

www.visionias.in
ANSWERS & EXPLANATIONS
GENERAL STUDIES (P) TEST – 2983 (2020)

Q 1.C
• Fixed-Rate or Fixed Float: Under this system, Central bank decides the official exchange rate. A set
price will be determined against a major world currency (usually the U.S. dollar, but also other major
currencies such as the euro, the yen, or a basket of currencies). To maintain the local exchange rate, the
central bank buys and sells its currency on the foreign exchange market in return for the currency to which
it is pegged.
• Flexible Rate or Free Float: When the exchange rate is decided by the market force (demand and supply
of currency), it is called the flexible exchange rate.
• Managed floating exchange rate: It is a combination of the flexible exchange rate system and a fixed
rate system. Under this, central banks sometimes intervene by selling foreign currencies in the exchange
to stabilize the domestic currency, this is called as dirty floating.

Q 2.C
• Marginal Standing Facility (MSF) rate refers to the rate at which the scheduled banks can borrow funds
overnight from RBI against government securities. It is a very short term borrowing scheme for scheduled
commercial banks. Banks may borrow funds through MSF during severe cash shortage or acute shortage
of liquidity. Hence statement 1 is not correct.
• Under MSF, a bank can borrow one-day loans form the RBI, even if it does not have any eligible
securities in excess of its SLR requirement (maintains only the SLR). Banks can borrow from the RBI up
to 1 % of their Net Demand and Time Liabilities or liabilities (or deposits) under MSF (increased to 2%
later). Hence statement 2 is not correct.
• All Scheduled Commercial Banks having Current Account and Subsidiary General Ledger Account
(SGL) with Reserve Bank, will be eligible to participate in the MSF Scheme. With effect from August
20, 2018 (RBI circular) LAF will also be extended to Scheduled State Co-operative Banks (StCBs)
which are Core Banking System (CBS) enabled and have Capital to Risk-Weighted Asset Ratio
(CRAR) of at least 9 percent. Hence statement 3 is not correct.

Q 3.C
• The Union Ministry of Health and Family Welfare recently published a national policy for the treatment
of ‘rare diseases’. Among other measures, the policy intends to kickstart a registry of rare diseases,
which will be maintained by the Indian Council of Medical Research (ICMR). Hence statement 1 is
correct and statement 2 is not correct.
• Broadly, a ‘rare disease’ is defined as a health condition of low prevalence that affects a small number of
people when compared with other prevalent diseases in the general population. While there is no
universally accepted definition of rare diseases, countries typically arrive at their descriptions,
taking into consideration disease prevalence, its severity and the existence of alternative therapeutic
options.
• Statement 3 is not correct: India does not have a definition of rare diseases because there is a lack of
epidemiological data on its incidence and prevalence. One of the immediate measures of this policy
is to arrive at a definition of rare diseases suited to India. Thus, the Central Committee will come up
with a definition of rare diseases for this policy, which could be revised in conformity with increased
information and knowledge about rare diseases.
• According to the policy, some of the rare diseases in India include genetic diseases, rare cancers,
infectious tropical diseases, and degenerative diseases. As per the policy, out of all rare diseases in the
1 www.visionias.in ©Vision IAS
world, less than five percent have therapies available to treat them. In India, roughly 450 rare diseases
have been recorded from tertiary hospitals, of which the most common are Haemophilia, Thalassemia,
Sickle-cell anemia, auto-immune diseases, Gaucher’s disease, and cystic fibrosis, etc.
• Under the policy, there are three categories of rare diseases — requiring one-time curative treatment,
diseases that require long-term treatment but where the cost is low, and those needing long-term
treatments with high cost. As per the policy, the assistance of Rs 15 lakh will be provided to patients
suffering from rare diseases that require a one-time curative treatment under the Rashtriya Arogya
Nidhi scheme. The treatment will be limited to the beneficiaries of Pradhan Mantri Jan Arogya Yojana.

Q 4.D
• The factor of production is "an economic term that describes the inputs that are used in the
production of goods or services to make an economic profit".
• The factors of production include land, labor, capital, and entrepreneurship. Out of these four,
entrepreneurship has recently been accepted as a potential new factor of production. Money is not a
factor of production.
• Money is only a facilitator in the acquisition of those goods. Money facilitates trade and stores value very
effectively, individuals cannot eat, wear or be sheltered by their bank account balances. The ultimate aim
of economic activity, work, and trade, is to acquire goods, not money.

Q 5.D
• All the statements are correct.
• According to the RBI, a wilful default is deemed to have occurred in any of the following four
circumstances:
o The unit has defaulted in meeting its payment/repayment obligations to the lender even when it
has the capacity to honour the said obligations.
o The unit has defaulted in meeting its payment/repayment obligations to the lender and has not
utilized the finance from the lender for the specific purposes for which finance was availed of
but has diverted the funds for other purposes.
o The unit has defaulted in meeting its payment / repayment obligations to the lender and has siphoned
off the funds so that the funds have not been utilised for the specific purpose for which finance was
availed of, nor are the funds available with the unit in the form of other assets.
o The unit has defaulted in meeting its payment / repayment obligations to the lender and has also
disposed off or removed the movable fixed assets or immovable property given for the purpose of
securing a term loan without the knowledge of the bank / lender (or the asset bought by the
lenders’ funds have been sold off without the knowledge of the bank/lender.)

Q 6.B
• Statement 1 is not correct: The Brus are spread across Tripura, Mizoram and parts of southern
Assam--are the most populous tribe in Tripura. Also known as Reangs in the state, they are ethnically
different from the Mizos, with their own distinct language and dialect and form one of the 21 scheduled
tribes of Tripura. While Mizoram has as many as 40,000 Brus living in the state, in Tripura, their numbers
stand at approximately 32,000, spread across six refugee camps in the state.
• Statement 2 is correct: Recently, displaced Bru tribals from Mizoram, living as refugees in Tripura since
1997, were allowed to permanently settle in Tripura. The agreement, allowing 30,000 Bru tribals to
permanently settle in Tripura was signed between the Centre, the state governments of Tripura and
Mizoram, and Bru-Reang representatives.
• In July 2018, a “four- corner agreement” was signed with provisions such as a one-time financial
assistance of Rs 4 lakh as fixed deposit in the name of the head of family only after 3 years of
uninterrupted stay in Mizoram, a house building assistance of Rs 1.5 lakh in three instalments and free
ration for two years and a monthly assistance of Rs 5,000 for each family. But it could not be
implemented could not be implemented as most refugees were not willing to go back to Mizoram
• In 1997, roughly half the Bru population fled to Tripura, following violent clashes with the Mizo
population, which led to the Brus’ demand for an Autonomous District Council (ADC), under the 6th
Schedule of the Constitution, in western Mizoram, where they were the more dominant lot, outnumbering
the ethnic Mizo population.

2 www.visionias.in ©Vision IAS


Q 7.B
• Increase in exports positively affects the exchange rate. Increase in exports of a country will lead to an
increase in demand for the currency and thus its value rises. Hence option 1 is not correct.
• Increase in imports will lead to higher demand for foreign currency and hence the demand for domestic
currency reduces. This will negatively affect the exchange rate. Hence option 2 is correct.
• Lower interest rate will attract less foreign investors to invest in the country and thus the demand
for currency will decrease, resulting in depreciation in value of the currency. Hence option 3 is correct.
• Higher inflation rate will make the country uncompetitive in the international market. The exports
will fall resulting in decreased demand for the currency and hence lower value. Hence option 4 is
correct.

Q 8.D
• A Non-Banking Financial Company (NBFC) is a company registered under the Companies Act, 1956 or
2013 engaged in the business of loans and advances, acquisition of shares, leasing, hire-purchase,
insurance business, chit business etc. NBFCs lend and make investments and hence their activities are
akin to that of banks. However, they are different from the Banks on the following parameters -:
o Acceptance of Demand Deposits
▪ Banks can do almost all financial services and products generally authorized to them. They can
accept demand deposits (demand deposits have high liquidity and is considered as good as
money). NBFCs can’t accept demand deposits.
o Part of the Payment and Settlement System
▪ While the Banks are part of the payment and settlement system, NBFCs do not form part of the
payment and settlement system and cannot issue cheques drawn on itself.
o The purview of deposit insurance facility of Deposit Insurance and Credit Guarantee
Corporation
▪ The deposit insurance facility of Deposit Insurance and Credit Guarantee Corporation is not
available to depositors of NBFCs, unlike in case of banks.
• Hence option (d) is the correct answer.

Q 9.B
• The Human Development Index (HDI) is a statistic composite index of life expectancy, education,
and per capita income indicators, which are used to rank countries into four tiers of human
development. It was developed by Pakistani economist Mahbub ul Haq. It is used to measure a country's
development by the United Nations Development Programme (UNDP).
• The HDI is the geometric mean of normalized indices for each of the three dimensions. The health
dimension is assessed by life expectancy at birth; the education dimension is measured by mean of
years of schooling for adults aged 25 years and more and expected years of schooling for children of
school entering age. The standard of living dimension is measured by gross national income per
capita. The HDI uses the logarithm of income, to reflect the diminishing importance of income with
increasing GNI. The scores for the three HDI dimension indices are then aggregated into a composite
index using geometric mean. Hence option (b) is the correct answer.

• The HDI simplifies and captures only part of what human development entails. It does not reflect on
inequalities, poverty, human security, empowerment, etc. The Human Development Report Office
(HDRO) offers the other composite indices as broader proxy on some of the key issues of human
development, inequality, gender disparity and poverty.

3 www.visionias.in ©Vision IAS


Q 10.A
• Futures are financial contracts obligating the buyer to purchase an asset or the seller to sell an asset, such
as a physical commodity or a financial instrument, at a predetermined future date and price. Futures
contracts detail the quality and quantity of the underlying asset; they are standardized to facilitate trading
on a futures exchange. Some futures contracts may call for physical delivery of the asset, while others are
settled in cash. Hence, statement 1 is correct. A futures exchange or futures market is a central
financial exchange where people can trade standardized futures contracts
• A forward contract is also a customized contract between two parties to buy or sell an asset at a specified
price on a future date. Unlike Futures Contracts, Forward contracts do not trade on a centralized exchange
and are therefore regarded as over-the-counter (OTC) instruments. Hence, statement 2 is not correct.

Q 11.C
• The Finance Commission is a Constitutionally mandated body that is at the centre of fiscal federalism.
Set up under Article 280 of the Constitution, its core responsibility is to evaluate the state of finances of
the Union and State Governments, recommend the sharing of taxes between them, lay down the principles
determining the distribution of these taxes among States.
• The Fifteenth Finance Commission was constituted in November 2017 against the backdrop of the
abolition of Planning Commission (as also of the distinction between Plan and non-Plan expenditure) and
the introduction of the goods and services tax (GST), which has fundamentally redefined federal fiscal
relations.
• The Terms of Reference of the current Commission have some distinctive features including
o The Commission shall use the population data of 2011 while making its recommendations. Hence
statement 1 is not correct.
o The Commission shall review the current status of the finance, deficit, debt levels, cash balances
and fiscal discipline efforts of the Union and the States, and recommend a fiscal consolidation
roadmap for sound fiscal management guided by the principles of equity, efficiency and transparency.
o The impact on the fiscal situation of the Union Government of substantially enhanced tax devolution
to States following recommendations of the 14th Finance Commission, coupled with the
continuing imperative of the national development programme including New India - 2022.
o The Commission may examine whether revenue deficit grants be provided at all.
o The Commission shall recommend performance-based incentives to the states depending on
parameters such as
▪ Efforts made in expansion of GST tax-net
▪ Efforts made in achieving replacement level of population growth
▪ Achievements in implementation of flagship schemes of Government of India, disaster resilient
▪ infrastructure
▪ Progress made in increasing capital expenditure, eliminating losses of power sector
▪ Progress made in promoting ease of doing business. Hence statement 3 is correct.
▪ Control or lack of it in incurring expenditure on populist measure
• Recently the Union Cabinet has approved the amendment to enable Fifteenth Finance Commission to
address serious concerns regarding the allocation of adequate, secure and non-lapsable funds for
defense and internal security of India. Hence statement 2 is correct.

Q 12.B
• A negotiable instrument is a signed document that promises a sum of payment to a specified person or the
assignee. In other words, it is a transferable, signed document that promises to pay the bearer a sum of
money at a future date or on-demand. The payee, who is the person receiving the payment, must be named
or otherwise indicated on the instrument.
• In India, the negotiable instruments are regulated by the Negotiable Instrument Act, 1881 as amended
from time to time. The different types of negotiable instruments recognized by the act include -:
o Promissory note: An instrument in writing containing an unconditional undertaking, signed by the
maker, to pay a certain sum of money only to or to the order of a certain person or the bearer of the
instrument. There are two parties, i.e. drawer and payee.
o Bill of exchange (demand draft): It is a written instrument showing the indebtedness of a buyer
towards the seller of goods. There are three parties, i.e. drawer, drawee and payee. Example- a student

4 www.visionias.in ©Vision IAS


paying a college fee through demand draft. Here, Student is a drawer, Bank is drawee and College is
the payee.
o Cheque: It is an order by the account holder of the bank directing his banker to pay on demand the
specified amount, to or to the order of the person named therein or to the bearer.
o Currency Note: It is a legal tender which gives the holder the right to receive the value of its
denomination on exchange. So “holding” determines the entitlement. Even if somebody steals the
currency he/she becomes the holder and so entitled to the rights. On the other hand, a negotiable
instrument is a document which guarantee a specific sum of money to the payee whose name is
mentioned in the instrument itself.
• Hence only options 1, 2 and 4 are correct.

Q 13.C
• Devaluating a currency is decided by the government issuing the currency, and unlike depreciation, is not
a result of non-governmental activities.
• Increased Aggregate Demand (AD) - Exports become cheaper and more competitive to foreign buyers.
Higher exports relative to imports can increase aggregate demand as increased consumer spending on
domestic goods and services.
• Inflation is more likely to occur because imports are more expensive causing cost-push inflation, AD is
increasing causing demand-pull inflation and with exports becoming, cheaper manufacturers may have
less incentive to cut costs and become more efficient. Therefore over time, costs may increase.
• Improvement in the current account balance. With exports more competitive and imports more
expensive, we may see higher exports and lower imports, which will reduce the current account deficit.
• Hence option (c) is the correct answer.

Q 14.D
• National Cyber Crime Reporting Portal (www.cybercrime.gov.in) is a citizen-centric initiative that
will enable citizens to report cyber crimes online through the portal.
• All the cyber crimes related complaints will be accessed by the concerned law enforcement agencies in
the States and Union Territories for taking action as per law.
• This portal was launched on a pilot basis on 30th August 2019 and it enables filing of all cyber crimes
with a specific focus on crimes against women, children, particularly child pornography, child sex
abuse material, online content related to rapes/gang rapes, etc.
• So far, more than 700 police districts and more than 3,900 police stations have been connected with
this Portal. After successful completion, this portal can improve the capacity of law enforcement
agencies to investigate the cases and will improve success in prosecution.
• This portal also focuses on crimes like financial crime and social media related crimes like stalking, cyber
bullying, etc. This portal will improve coordination amongst the law enforcement agencies of different
States, districts and police stations for dealing with cyber crimes in a coordinated and effective manner.
MHA is committed to provide and create an ecosystem for dealing with the cyber crimes in a
comprehensive & coordinated manner.

Q 15.A
• Primarily, banks offer two kinds of deposit accounts. These are demand deposits like current/saving
account and term (or time) deposits like fixed or recurring deposits. When you open a deposit account
in a bank, you become an account holder or a depositor. Thus saving deposits and current account
deposits are types of demand deposits. Demand deposit funds deposited in a bank account at a low or
a zero interest rate, which allows depositors to directly withdraw their money and issue bank checks up
to the limit of their account balance at any time. Hence option 1 is correct.
• Term Deposits are one of the best investment options for people who are looking for a stable and safe
return on their investments. In Term Deposits, the sum of money is kept for a fixed maturity and the
depositor is not allowed to withdraw this sum till the end of the maturity period. That is why they are
called Term Deposits because they are kept up to a particular term. But when it comes to Term Deposit,
here’s what you need to know.
• There are two types of Term Deposits:
• Recurring Deposits- In a Recurring Deposit, a fixed sum of money is invested at a fixed interval. In most
cases, this interval is once a month. The investments earn interest on them till the maturity period. To put

5 www.visionias.in ©Vision IAS


it simply, a Recurring Deposit is like opening several different Fixed Deposits, each with the same
maturity period. Hence option 2 is not correct.
• Fixed Deposits- Fixed Deposits are deposits where a particular sum of money is invested for a fixed
duration. The duration of Fixed Deposits is flexible. It can range from 7 days to 10 years. The rate of
interest for the Fixed Deposit depends on the period for which the funds are locked in. Hence option 3 is
not correct.

Q 16.A
• Pigovian tax is a special tax that is often levied on companies that pollute the environment or create
excess social costs, called negative externalities, through business practices. In a true market economy, a
Pigovian tax is the most efficient and effective way to correct negative externalities. Tax on diesel, tax on
tobacco products and alcohol are examples of Pigouvian Tax.

Q 17.D
• Liquidity Adjustment Facility (LAF) is a monetary policy tool of Reserve Bank of India for modulating
liquidity and transmitting interest rate signals to the market. It is used to aid banks in adjusting the day to
day mismatches in liquidity. The two components of LAF are repo rate and reverse repo rate. Under Repo,
the banks borrow money from RBI to meet short term needs by putting government securities as
collateral. Under Reverse Repo, RBI borrows money from banks by lending securities.
• Other than Regional Rural Banks, all the Scheduled Commercial Banks are eligible to participate
in auctions. Primary Dealers (PDs) and NBFCs are also eligible to participate in auctions.
• Primary dealers are registered entities with the RBI who have the license to purchase and sell government
securities. They are entities who buy the government securities directly from the RBI (the RBI issues
government securities on behalf of the government), aiming to resell them to other buyers.
• Hence (d) is the correct answer.

Q 18.B
• Foreign exchange reserves are an important component of the balance of payments and an essential
element in the analysis of an economy’s external position. The components of India’s foreign exchange
reserves are foreign currency assets (FCA), gold, SDRs and reserve tranche position (RTP) in the
IMF. Foreign Currency Assets (FCA) is the biggest component of the forex reserves. Hence statement 1
is not correct.
• Forex reserves act as insurance when the rupee tends to be volatile against the dollar. In the process of
supporting weakening rupee, the RBI needs to buy dollars, ultimately, leading to higher forex buildups.
But when RBI purchases dollars, it leads to infusion of rupee into the system which leaves an inflationary
effect on the economy. An increase in foreign exchange could be because of buying dollars by the RBI
which leads to an increase in the money supply. Hence statement 2 is correct.

Q 19.B
• Skewflation means the skewness of inflation among different sectors of the economy - some goods and
services are facing huge inflation, some none and some deflation. Hence option b is the correct
answer.
• In India, food prices rose steadily during the last months of 2009 and the early months of 2010, even
though the prices of non-food items continued to be relatively stable. As this somewhat unusual
phenomenon stubbornly persisted, and policymakers conferred on how to bring it to an end, the term
‘skewflation’ made an appearance in the Economic Survey 2009-10, Government of India, Ministry of
Finance.
• Given that it is sector-specific, it is not evident that it calls for monetary or fiscal policy action. On the
other hand, given its sustained nature, it is not possible for the government to ignore it since causes stress
to consumers.
• Stagflation: Stagflation, or recession-inflation, is a situation in which the inflation rate is high, the
economic growth rate slows, and unemployment remains steadily high. It presents a dilemma for
economic policy, since actions intended to lower inflation may exacerbate unemployment, and vice versa.
6 www.visionias.in ©Vision IAS
Q 20.B
• Liberalization of trade and investment regime was initiated under the New Economic Policy to increase
the international competitiveness of industrial production and also foreign investments and technology
into the economy. The aim was also to promote the efficiency of local industries and the adoption of
modern technologies.
• To protect the domestic industries, India was following a regime of quantitative restrictions on imports.
This was encouraged through tight control over imports and by keeping the tariffs very high. These
policies reduced efficiency and competitiveness which led to the slow growth of the manufacturing sector.
• The trade policy reforms aimed at:
o Dismantling of quantitative restrictions on imports and exports. (An import quota is a type of trade
restriction that sets a physical limit on the quantity of a good that can be imported into a country in the
given period of time)
o Quantitative restrictions on imports of manufactured consumer goods and agricultural products were
also fully removed from April 2001. Hence option 3 is not correct.
o Reduction of tariff rates. Export duties have been removed to increase the competitive position of
Indian goods in the international markets. Hence option 1 is correct.
o Removal of licensing procedures for imports. Import licensing was abolished except in case of the
hazardous and environmentally sensitive industries. Hence option 2 is correct.

Q 21.A
• The Ministry of Finance concerns itself with taxation, financial legislation, financial institutions, capital
markets, centre and state finances, and the Union Budget. The Union Finance Ministry of India
comprises five departments.
• Department of Expenditure: It is the nodal Department for overseeing the public financial
management system in the Central Government and matters connected with state finances. It is
responsible for the implementation of the recommendations of the Finance Commission and Central
Pay Commission, monitoring of audit comments/ observations, preparation of Central Government
Accounts. Hence pair 1 is correctly matched.
• Department of Revenue: It functions under the overall direction and control of the Secretary (Revenue).
It exercises control in respect of matters relating to all the Direct and Indirect Union Taxes through two
statutory Boards namely, the Central Board of Direct Taxes (CBDT) and the Central Board of Indirect
Taxes and Customs (CBIC).
• Department of Economic Affairs: It is responsible for advice on economic issues having a bearing
on internal and external aspects of the Indian economy including inflation, price control, foreign
exchange management, Official Development Assistance domestic finance and preparation of the
Union Budget, bilateral and multilateral engagement with international financial institutions and other
countries. The Department is also the parent cadre of the Indian Economic Service. Hence pair 2 is not
correctly matched.
• Department of Financial Services: The mandate of the Department of Financial Services covers
the functioning of Banks, Financial Institutions, Insurance Companies and, the National Pension
System. The Department of Financial Services (DFS) oversees several key programs/initiatives and
reforms of the Government concerning the Banking Sector, the Insurance Sector and the Pension Sector in
India. Initiatives and reforms relating to Financial Inclusion, Social Security, and Insurance as a Risk
Transfer mechanism; Credit Flow to the key sectors of the economy/ farmers/ common man are some of
the key focus areas being dealt by the Department. Hence pair 3 is correctly matched.
• Department of Investment and Public Asset Management (DIPAM): As per the present Allocation of
Business Rules, the mandate of the Department is as follows:
o All matters relating to the management of Central Government investments in equity including
disinvestment of equity in Central Public Sector Undertakings.
o All matters relating to the sale of Central Government equity through an offer for sale or private
placement or any other mode in the erstwhile Central Public Sector Undertakings.
• Hence option (a) is the correct answer.

7 www.visionias.in ©Vision IAS


Q 22.C
• The income elasticity of demand is a measure of the relationship between a change in the quantity
demanded for a particular good and a change in real income. It refers to the sensitivity of the quantity
demanded for a certain product in response to a change in consumer incomes.
• The formula for calculating income elasticity of demand is: Income Elasticity of Demand = % change in
quantity demanded / % change in income
• Normal goods have a positive income elasticity of demand. As incomes rise, more goods are demanded
at each price level. The quantity demanded for normal necessities will increase with income, but at a
slower rate than luxury goods. This is because consumers, rather than buying more of the necessities, will
likely use their increased income to purchase more luxury goods and services. During a period of
increasing incomes, the quantity demanded for luxury products tends to increase at a higher rate than the
quantity demanded for necessities. The quantity demanded for luxury goods is very sensitive to changes in
income.
• Inferior goods have a negative income elasticity of demand - the quantity demanded for inferior goods
falls as incomes rise. For example, the quantity demanded for generic food items tends to decrease during
periods of increased incomes.
• Hence, both the given statements are correct.

Q 23.A
• Before a loan account turns into a Non –Performing Asset (NPA), banks are required to identify incipient
stress in the account by creating three sub-categories under the Special Mention Account (SMA) category
as given in the table below:
• SMA Sub categories:
o SMA-0: Principal or interest payment not overdue for more than 30 days but account showing signs
of incipient stress.
o SMA-1: Principal or interest payment overdue between 31-60 Days.
o SMA-2: Principal or interest payment overdue between 61-90 Days.
• Categories of NPAs
o Banks are required to classify non-performing assets further into the following three categories based
on the period for which the asset has remained non-performing and the realisability of the dues:
▪ Substandard Assets
▪ Doubtful Assets
▪ Loss Assets
• Substandard Assets: With effect from March 31, 2005, a sub-standard asset is identified as one, which
has remained NPA for a period less than or equal to 12 months. Such an asset will have well-defined
credit weaknesses that jeopardize the liquidation of the debt and are characterized by the distinct
possibility that the banks will sustain some loss if deficiencies are not corrected.
• Doubtful Assets: With effect from March 31, 2005, an asset is classified as doubtful if it has remained in
the sub-standard category for a period of 12 months. A loan classified as doubtful has all the weaknesses
inherent in assets that were classified as sub-standard, with the added characteristic that the weaknesses
make collection or liquidation in full, - based on currently known facts, conditions and values – highly
questionable and improbable.
• Loss Assets: A loss asset is one where loss has been identified by the bank or internal or external auditors
or the RBI inspection but the amount has not been written off wholly. In other words, such an asset is
considered uncollectible and of such little value that its continuance as a bankable asset is not warranted
although there may be some salvage or recovery value.

Q 24.A
• Chinese paddlefish is one of the world's largest freshwater fish, with some individuals reaching a mind-
boggling 23 feet (7 meters) in length and weighing half a ton (450 kilograms). They have long, silver-gray
bodies, very large mouths, and a long, wide snout that resembles a paddle. The snouts contain sensors that
help them locate the small fish and crustaceans they survive on.
• As recently as the 1970s, fishermen harvested 25 tons of Chinese paddlefish from the Yangtze River. By
the 1990s, almost none remained. The last official sighting in the wild was in 2003, and just as 2019
turned to 2020, scientists made it official: The Chinese paddlefish is extinct. It was often called the
“Giant Panda of the Rivers” in recognition of its rarity. It is important to note here that Paddlefish is

8 www.visionias.in ©Vision IAS


also found in the Mississippi river basin along with Yangtze river basin. American paddlefish are found in
Mississippi river delta, Mississippi and Missouri river tributaries. It is the Chinese paddlefish that has
been declared extinct.
• Arapaima is native to the Amazon basin and among the world’s largest freshwater fish. The species of
arapaima is native to Amazon river basin and air breather – pirarucu.
• Mekong Giant Catfish is the largest catfish in the world, native to Mekong river basin in Southeast Asia.
This Mekong river catfish is on the verge of extinction due to overfishing.
• Beluga Sturgeon fish are the largest freshwater fish in the world followed by White sturgeons from the
Columbia River in North America. Sturgeon spend most of their lives in freshwater environments, river
deltas, and estuaries.
Q 25.A
• CIC or Credit Information Company is an independent third party institution that collects financial data
regarding loans, credit cards and more about individuals and shares it with its members. Banks, Non-
Banking Financial institutions are usually the customers of Credit Information Companies. The Credit
Card Company collects financial information about all these individuals and forms a credit report based
on their financial history. This credit report plays a very important role as it helps banks and other
financial institutions determine the creditworthiness of an individual applying for a loan or credit card
with them. Hence statement 1 is correct.
• Whenever an individual approaches a bank/financial institution to avail loans, the bank will be concerned
about the repayment capacity of the individual. The repayment capacity can be traced from the loan
repayment history of the individual.
• At present, four credit information companies are given a certificate of registration by the Reserve Bank
of India. These companies are Credit Information Bureau (India) Limited (CIBIL), Equifax Credit
Information Services Private Limited, Experian Credit Information Company of India Private Limited and
CRIF High Mark Credit Information Services Private Limited.
• For the regulation of credit information activities, there is the Credit Information Companies (Regulation)
Act, 2005 (CICRA) which is binding for the Credit information companies as well as for financial
institutions. The CICRA was enacted to regulate the businesses of the CICs. Hence statement 2 is not
correct.
Q 26.D
• Wholesale Price Index (WPI) measures the average change in the prices of commodities for bulk sale at
the level of the early stage of transactions. The index basket of the WPI covers commodities falling under
the three major groups namely Primary Articles, Fuel and Power and Manufactured Products.
• The prices tracked are ex-factory prices for manufactured products, mandi price for agricultural
commodities and ex-mines prices for minerals. Weights given to each commodity covered in the WPI
basket is based on the value of production adjusted for net imports. WPI basket does not cover services.
• In India, WPI is also known as the headline inflation rate. In India, Office of Economic Advisor (OEA),
Department for Promotion of Industry and Internal Trade (DPIIT), Ministry of Commerce and Industry
calculates the WPI. Hence statement 1 is not correct.
• While the direction of the Consumer Price Index is often in sync with the direction of WPI, there may be a
divergence as well. The reasons for the divergence between the two indices can also be partly attributed to
the difference in the weight of the food group in the two baskets. CPI Food group has a weight of 39.1
percent as compared to the combined weight of 24.4 percent (Food articles and Manufactured Food
products) in WPI basket. The CPI basket consists of services like housing, education, medical care,
recreation etc. which are not part of WPI basket. A significant proportion of WPI item basket represents
manufacturing inputs and intermediate goods like minerals, basic metals, machinery, etc. whose prices are
influenced by global factors but these are not directly consumed by the households and are not part of the
CPI item basket. Thus even significant price movements in items included in WPI basket need not
necessarily translate into movements in CPI in the short run. Hence statement 2 is not correct.

Q 27.A
• Disinvestment means the dilution of the stake of the Government in a public enterprise. Strategic
disinvestment is transferring the ownership and control of a public sector entity to some other entity
(mostly to a private sector entity). Unlike the simple disinvestment, strategic sale implies some sort of
privatization.
9 www.visionias.in ©Vision IAS
• The Department of Investment and Public Asset Management (DIPAM) under the Ministry of
Finance has been made the nodal department for the strategic stake sale in the Public Sector Undertakings
(PSUs). Hence statement 1 is not correct.
• Types of Disinvestments are:
o Disinvestment through minority stake sale: While pursuing disinvestment through minority stake
sale in listed CPSEs, the Government will retain majority shareholding, i.e. at least 51 percent of the
shareholding and management control of the Public Sector Undertakings. The Department of
Investment and Public Asset Management (DIPAM) is to identify CPSEs in consultation with
respective administrative Ministries and submit a proposal to Government in cases requiring Offer for
Sale of Government equity.
o Strategic Disinvestment: It is to be undertaken through a consultation process among different
Ministries/Departments, including NITI Aayog. NITI Aayog identifies CPSEs for strategic
disinvestment and advises on the mode of sale, percentage of shares to be sold of the CPSE and
method for valuation of the CPSE. Hence statement 2 is correct.
• Disinvestment proceeds in the last 5 years.
Disinvestment
Year
proceeds
2014-15 24,277.17
2015-16 23,996.80
2016-17 34,938.68
2017-18 57,273.05
2018-19 62,883.20
• From the above table, it is clear that Disinvestment proceeds though increased but there was no steady
pattern in its growth in the last 5 years. Hence statement 3 is not correct.
• Hence option a is the correct answer.

Q 28.D
• Stagflation is a blend of two words i.e. stagnation and inflation. It is used in economics when the
inflation rate is high, the growth rate slows down, and unemployment stays high. It raises a dilemma for
economic policy since actions designed to lower inflation may exacerbate unemployment and vice versa.
Hence option (d) is the correct answer.
• Hyperinflation (> 1000%)- This is reserved for extreme forms of inflation – usually over 1,000% though
there is no specific definition. Hyperinflation usually involves prices changing so fast, that it becomes a
daily occurrence, and under hyperinflation, the value of money will rapidly decline.
• Skewflation means the skewness of inflation among different sectors of the economy - some goods and
services are facing huge inflation, some none and some deflation.
• Disinflation is a decrease in the rate of inflation – a slowdown in the rate of increase of the general
price level of goods and services in a nation's gross domestic product over time. It is the opposite of
reflation.

Q 29.B
• Inflation is a sustained increase in the general price level of goods and services in an economy over a
period of time. It is caused by an increase in the money supply in the economy. It is a situation where too
much of money is chasing fixed number of goods and services.
• Money supply: If the volume of goods and services produced in the economy remains unchanged, the
extra money will lead to an increase in prices of all commodities. People have more money in their
hands with which they compete each other (increase in aggregate demand) in the commodities market
for buying the same old stock of goods. As too much money is now chasing the same old quantities of
output, the process ends up in bidding up prices of every commodity - an increase in the general price
level, which is also known as inflation. Hence increase in money supply and aggregate demand
increases the price levels in an economy.
• When the RBI wants to increase the money supply in the economy, it purchases the government
securities from the market and it sells government securities to suck out liquidity from the

10 www.visionias.in ©Vision IAS


system. Hence Sale of G-Secs will decrease the money supply and has the effect of decreasing the
price levels.
• Hence option (b) is the correct answer.

Q 30.B
• The introduction of General Anti-Avoidance Rules (GAAR) is a watershed event in the evolution of
India’s tax policy and legislation. GAAR is an anti-tax avoidance law under Chapter X-A of the
Income Tax Act, 1961 of India. It is framed by the Department of Revenue under the Ministry of
Finance. GAAR has been made effective from 1 April 2017. Hence option b is the correct answer and
option c is not the correct statement.
• GAAR applies to any arrangement that is considered an Impermissible Avoidance Arrangement
(IAA). Furthermore, under its provisions, certain transactions are deemed to lack commercial
substance. GAAR is not merely restricted to cross-border transactions, but also applies to domestic
arrangements. Once the Revenue authorities decide to treat an arrangement as an IAA, the onus to prove
otherwise is on taxpayers. Consequently, they are required to substantiate the commercial reasons for such
arrangements and that availing tax benefit was not the main purpose for these transactions. Hence option
a is not the correct statement.
• Under the Income Tax act, a taxpayer has the right to appeal to the Income Tax Appellate Tribunal
(ITAT) against an order passed by the Revenue authorities along with the direction of the Approving
Panel. (A subsequent appeal may be filed before a High Court and the Supreme Court.). Hence
option d is not the correct statement.

Q 31.A
• Domestic Systemically Important Bank (D-SIB) means that the bank is too big to fail. These are
recognized by the Reserve Bank of India. According to the RBI, some banks become systemically
important due to their size, cross-jurisdictional activities, complexity and lack of substitute and
interconnection. Banks whose assets exceed 2% of GDP are considered part of this group. Hence
statement 1 is correct and statement 2 is not correct.
• The RBI stated that should such a bank fail, there would be significant disruption to the essential services
they provide to the banking system and the overall economy. The too-big-to-fail tag also indicates that in
case of distress, the government is expected to support these banks. Due to this perception, these banks
enjoy certain advantages in funding. It also means that these banks have a different set of policy measures
regarding systemic risks and moral hazard issues.
• As per the framework, from 2015, every August, the central bank has to disclose the names of banks
designated as D-SIB. It classifies the banks under five buckets depending on the order of importance.
Based on the bucket in which a D-SIB is, an additional common equity requirement applies.
• As of now, RBI has recognized, HDFC Bank (Private sector), ICICI (Private Sector) and SBI (Public
Sector) under the category of D-SIBs. Hence statement 3 is not correct.

Q 32.C
• Sovereign bonds are debt instruments created to raise capital. They are essentially loan agreements
between the bond issuer and an investor, in which the bond issuer is obligated to pay a specified amount
of money at specified future dates. Finance Minister Nirmala Sitharaman in her maiden Budget
speech said that the Indian Government would start raising a part of its gross borrowing program in
external markets in external currencies.
• There are many reasons why India is borrowing in external markets in external currency. Possibly the
biggest of these is that the Indian government’s domestic borrowing is crowding out private
investment and preventing the interest rates from falling even when inflation has cooled off and the
RBI is cutting policy rates. If the government was to borrow some of its loans from outside India, there
will be investable money left for private companies to borrow and interest rates could start coming
down. Hence statement 1 is correct and statement 2 is not correct.
• Also, a sovereign bond issue will provide a yield curve - a benchmark - for Indian corporates who wish
to raise loans in foreign markets. This will help Indian businesses that have increasingly looked towards
foreign economies to borrow money. Globally, and especially in the advanced economies where the
government is likely to go to borrow, the interest rates are low and, thanks to the easy monetary policies
of foreign central banks, there are a lot of surplus funds waiting for a product that pays more.

11 www.visionias.in ©Vision IAS


• However many economists expressed concerns about India’s sovereign bonds that will be floated in
foreign countries and will be denominated in foreign currencies. In other words, both the initial loan
amount and the final payment will be in either US dollars or some other comparable currency. If the loan
is in terms of dollars, and the rupee weakens against the dollar during the bond’s tenure, the
government would have to return more rupees to pay back the same amount of dollars. Hence
statement 3 is correct.

Q 33.C
• Coronaviruses are a large family of viruses with some causing less severe common cold to more severe
diseases such as the Severe acute respiratory syndrome (SARS) and Middle East respiratory syndrome
(MERS).
• Coronaviruses are zoonotic, meaning they are transmitted between animals and people. While SARS
coronavirus was transmitted from civet cats to humans in China in 2002, MERS coronavirus was from
dromedary camels to humans in Saudi Arabia in 2012. Hence, statements 1 and 2 are correct.
• A novel coronavirus has been identified in 41 people in Wuhan City, Hubei Province of China. On
January 9, 2020, the WHO issued a statement saying Chinese researchers have made “preliminary
determination” of the virus as a novel coronavirus in a person with pneumonia. The researchers were able
to determine the virus by sequencing the genome of the novel virus using an isolate taken from an infected
patient. Hence, statement 3 is not correct.

Q 34.A
• At least three public sector banks (Union Bank of India, Indian Bank and Central Bank of India) that
reported earnings for the January-March 2019 quarter mentioned ‘divergence’ in bad loan recognition and
have made provisions for such loans. Divergence takes place when the Reserve Bank of India (RBI) finds
that a lender has under-reported (or not reported at all) bad loans in a particular year and hence asks the
lender to make disclosures if under-reporting is more than 10% of bad loans or the provisioning. When a
loan is not being repaid, the Bank has to reconstitute this Money from its other sources like Profit. Setting
aside of money from Profits to compensate a probable loss caused on lending a loan is called
Provisioning.
• The provision coverage ratio (PCR) indicates the provision made against bad loans from the profit
generated. Higher the PCR, lower is the unexposed part of the bad debts. The PCR of public sector
banks has risen steeply from 46.04 per cent as of March 2015 to 66.85 per cent as of September 2018,
giving banks cushion to absorb losses. Hence option (a) is the correct answer.
• The asset coverage ratio is a risk measurement that calculates a company's ability to repay its debt
obligations by selling its assets. It provides a sense to investors of how much assets are required by a firm
to pay down its debt obligation.
• The collateral coverage ratio is equal to the total discounted collateral value divided by the total loan
request. Collateral refers to personal and business assets, such as a house, car, office equipment, truck and
heavy equipment, inventory, receivables, stocks, bonds and certificates of deposit.
• The liquidity coverage ratio (LCR) refers to the proportion of highly liquid assets held by financial
institutions, to ensure their ongoing ability to meet short-term obligations.

Q 35.C
• Cyclical unemployment is unemployment that results when the overall demand for goods and
services in an economy cannot support full employment. It occurs during periods of slow economic
growth or during periods of economic contraction. It relates to the irregular ups and downs, or cyclical
trends in growth and production, as measured by the gross domestic product (GDP), that occur within
the business cycle. Hence option c is the correct answer.
• When economic activity increases, we call this an expansionary phase of the business cycle because it
represents economic growth. Unemployment tends to drop during periods of growth because
consumers are buying more and businesses are producing and selling more. These increases in production
and sales usually require more people, resulting in increased hiring and an overall reduction of the rate of
unemployment.
• When economic activity slows, or contracts, unemployment will increase. During times of slow growth,
no growth or a period of economic contraction, a recession, demand for products and services declines.
Employers make less, sell less and don't need as many employees. Employers will lay off those
employees who are not needed, which raises the unemployment rate. This increase in unemployment
12 www.visionias.in ©Vision IAS
can lead to a problematic feedback loop where unemployed people can no longer afford to buy stuff,
which means that demand decreases even further and more people are laid off, leading to even less
demand and so on.

Q 36.D
• A 'bank run' occurs when a large number of customers of a bank or another financial institution withdraw
their deposits simultaneously due to concerns about the bank's solvency. As more people withdraw their
funds, the probability of default increases, thereby prompting more people to withdraw their deposits. In
extreme cases, the bank's reserves may not be sufficient to cover the withdrawals.
• When a bank cannot satisfy customer demands for withdrawals—or if there’s a rumor that the bank will
be unable to do so—the situation worsens. Customers fear being the “last one to the exit,” and they
attempt to withdraw as much as possible. In a worst-case scenario, a bank may be unable to meet
obligations, leading to complete failure.
• Hence option (d) is the correct answer.

Q 37.D

• The primary regulator for insurance in India is the Insurance Regulatory and Development Authority of
India (IRDAI) which was established in 1999 under the government legislation called the Insurance
Regulatory and Development Authority Act, 1999. Hence pair 1 is not correctly matched.
• Nidhi Company is a type of Non-Banking Financial Company (NBFC). It is formed to borrow and lend
money to its members. It inculcates the habit of savings among its members and works on the principle of
mutual benefit. Hence pair 2 is not correctly matched.
• A Collective Investment Scheme is any scheme or arrangement, which satisfies the conditions, referred
to in sub-section (2) of section 11AA of the SEBI Act. Any scheme or arrangement made or offered by
any company under which the contributions, or payments made by the investors, are pooled and utilized to
13 www.visionias.in ©Vision IAS
receive profits, income, produce or property, and is managed on behalf of the investors is a CIS. Investors
do not have day to day control over the management and operation of such a scheme or
arrangement. Hence pair 3 is not correctly matched.
• Hence option (a) is the correct answer.

Q 38.D
• Fiscal Drag means an adverse effect of progressive taxation on demand and economic expansion.
• It occurs when due to the rise in national income, it brings in more and more people under the tax net due
to progressive taxation and due to a rise in their incomes. However, while the nominal income has gone
up, but real income may not have correspondingly increased, as the higher taxation leads to a reduction in
aggregate demand which may bring in a slowdown in the economy due to low demand.
• It is automatic restraint applied to a fast-expanding economy under a progressive tax system. As incomes
rise, individuals are pushed into higher income tax brackets, and average tax obligations increase that
dampen spending.

Q 39.D
• The Priority Sector Lending Certificates (PSCLs) are certificates issued by banks that have
overreached their priority sector lending targets. Hence statement 1 is not correct. PSLCs thus can
be issued only up to the extent of their over lending to the stipulated sectors. Buyers of PSLCs are usually
those banks that could not meet their priority sector lending targets. The price of PSLCs will be
determined based on demand and supply that will be reflected in the auction under the RBI’s e-
Kuber trading platform.
• As per the RBI guidelines, banks can issue four types of PSLCs including three subsector PSLCs-
agriculture, small and marginal farmers, micro enterprises and one PSLC for general.
• All Scheduled Commercial Banks (including Regional Rural Banks), Urban Co-operative Banks, Small
Finance Banks (when they become operational) and Local Area Banks are eligible to participate in the
trading. However, Payment banks are not allowed to give any form of a loan or issue a credit card, which
is also a form of unsecured personal loan. Consequently, they are not within the purview of buying or
selling PSLCs. Hence statement 2 is not correct.

Q 40.C
• There are macro variables, a rise in the level of which leads to a fall in the level of output in the
economy. These include (i) saving (ii) imports (iii) government taxes. A rise in the level of any of these
variables implies a fall in the level of demand for the domestically produced goods. This leads to a fall in
the level of output in the economy.
• There are macro variables, a rise in the level of which leads to a rise in the level of output in the
economy. These include:
o government consumption expenditure
o government investment expenditure
o exports
• A rise in the level of any of these variables implies a rise in the level of demand for goods produced in the
domestic economy. This is expected to cause a rise in the level of output in the economy.

Q 41.D
• The Miyawaki method of afforestation, pioneered by Japanese botanist Akira Miyawaki, originated in
Japan and is now increasingly adopted in other parts of the world. By promoting natural vegetation on
land destroyed by natural calamities and man-induced mistakes, Miyawaki managed to raise mini forests
along the coastline of Japan. As a side benefit, these forests serve as a natural bulwark against soil erosion
and Tsunami.
• It has revolutionized the concept of urban afforestation by turning backyards into mini-forests. Using this,
it’s possible to grow a variety of native species in as little space as 600 sq.ft.
• It was recently proposed to be adopted on the government office premises, residential complexes, school
premises, and poramboke land (means unassessed lands which are the property of Government used or
reserved for public) in Kerala, and in various places in Tamil Nadu. Telangana government has introduced
the Japanese Miyawaki method of afforestation to grow urban forests and expand the green cover.

14 www.visionias.in ©Vision IAS


• In contrast to conventional planting techniques, this method allows for planting more number of trees in
small spaces. The trees grow faster too and are free of chemicals and fertilizers. The advantage of
Miyawaki method is that forests need to be maintained only for two years after which they can sustain
themselves.
The exercise will cost approximately ₹ 20,000 for 600 sq. ft mini forest.
• Some of the disadvantages of the technique are that the Forests are very dense, which restricts the
movement of any possible wildlife the forest might attract.

Q 42.D

• Hence statement 2 is not correct as per the above-given image.


• The difference between total revenue and total expenditure of the government is termed as fiscal
deficit. It is an indication of the total borrowings needed by the government.
Fiscal
Year
Deficit
2013-14 2.2
2014-15 2.6
2015-16 3.1
2016-17 3.5
2017-18 3.0
• As given in the above table, it can be concluded that the Fiscal deficit has steadily not increased in the last
5 years. Hence statement 1 is not correct.

Q 43.C
• Near money is an economic term describing non-cash assets that can be easily converted into cash
but are not used as a medium of exchange in everyday transactions. For example, the time deposits,
Bills of Exchanges, Government and Private Bonds Saving certificates, shares, etc. possess the power
of money, yet they are not used to perform immediate economic activities.
• Currency notes and demand deposits such as cheques, demand drafts, etc. are more liquid than near
money and can be used as a medium of exchange in everyday transactions. Thus they are not categorized
as near money.
• Real estate can not be called near money. The holder of the real estate needs to convert it into the
money before performing economic activities.
• Hence option (c) is the correct answer.

15 www.visionias.in ©Vision IAS


Q 48.D
• Among the most-discussed new concepts at the annual Consumer Electronics Show (CES) in Las Vegas
this year was NEON. The first project of Samsung’s Star Labs, NEONs is being called the world’s first
artificial humans. They look and behave like real humans, and could one day develop memories and
emotions — though from behind a 4K display. Currently, NEONs don’t have a physical embodiment, but
rather, are digitally composed next-generation artificial intelligent entities.
• NEONs are computationally created virtual humans — the word derives from NEO (new) +
humaN. For now, the virtual humans can show emotions when manually controlled by their creators. But
the idea is for NEONs to become intelligent enough to be fully autonomous, showing emotions, learning
skills, creating memories, and being intelligent on their own.

Q 49.C
• The Capital Budget is an account of assets as well as liabilities of the central government.
• Capital Receipts: All those receipts of the government which either creates liability or reduces financial
asset are capital receipts.
• Examples:
o Market borrowings by the government from the public,
o Borrowings from the RBI,
o Borrowings from commercial banks or financial institutions through the sale of Treasury-bills,
o Loans received from foreign governments or international financial institutions, post office savings,
post office saving certificates, and PSU’s Disinvestment.
• Capital Expenditure: All those expenditures of the government which either result in the creation of
physical/financial assets or reduction in financial liabilities.
• Examples:
o Purchase of land, machinery, building, and equipments;
o investment in shares;
o loans and advances by the central government to state governments and UTs.
• Interest paid on the loans taken by the Government is part of the revenue account of the Budget.
• Hence option (c) is the correct answer.

Q 50.A
• NASA’s Transiting Exoplanet Survey Satellite (TESS) has discovered its first Earth-size planet in its
star’s habitable zone, the range of distances where conditions may be just right to allow the presence of
liquid water on the surface. Scientists confirmed the find, called TOI 700 d, using NASA’s Spitzer Space
Telescope and have modeled the planet’s potential environments to help inform future observations.
• TOI 700 d is one of only a few Earth-size planets discovered in a star's habitable zone so far. Others
include several planets in the TRAPPIST-1 system and other worlds discovered by NASA’s Kepler Space
Telescope.
• TOI 700 is a small, cool M dwarf star located just over 100 light-years away in the southern constellation
Dorado. It’s roughly 40% of the Sun’s mass and size and about half its surface temperature.

Q 51.D
• The government has recently launched a central accident database management system that will
help in analyzing causes of road crashes and in devising safety interventions to reduce such
accidents in the country. This project is proposed on IT-based system for capturing the spot
accident data using mobile app and tablets configured for this purpose. This data can then be utilized
for various purposes like finding the causes of the accidents and remedial measures to improve the road
infrastructure, to record the accident data for the use of police, health services, and other concerned
departments.
• The IT tool, known as the Integrated Road Accident Database (IRAD), has been developed by the Indian
Institute of Technology-Madras (IIT-M) and will be implemented by the National Informatics
Centre. The project costs ₹258 crore and is being supported by the World Bank. Around 30,000 tablets
would be distributed to police personnel across the country to record data on accidents.
• The system will be first piloted in the six States with the highest fatalities from road crashes — Karnataka,
Madhya Pradesh, Maharashtra, Rajasthan, Tamil Nadu, and Uttar Pradesh.

17 www.visionias.in ©Vision IAS


• India sees the largest number of road fatalities in the world. More than 1.5 lakh people lost their lives in
road crashes in the country in 2018, according to government data. Of the total people killed in road
crashes in 2018, 48% were between 18 years and 35 years old, and more than 60% of such fatalities were
due to overspeeding.
• Hence both the statements are correct.

Q 52.B
• Zero-based budgeting (ZBB) is a method of budgeting in which all expenses must be justified for
each new period. The process of zero-based budgeting starts from a "zero base," and every scheme
within the budget is analyzed for its needs and costs. Budgets are then built around what is needed for the
upcoming period, regardless of whether each budget is higher or lower than the previous one. The
primary purpose of zero-based budgeting is the termination of activities which have become
irrelevant.
• Advantages of Zero-Based Budgeting
o It leads to efficiency. It works on rational principles leading to the efficient allocation of resources
(department-wise).
o It brings accuracy. In traditional budgeting, some arbitrary changes are made to the previous year’s
budget. But in the ZBB framework, every department relooks at every item of the cash flow and
accordingly computes its operating costs. It gives a clear idea of the costs involved against the desired
performance. It also assists in cost reduction to a limited extent.
o It minimizes redundant activities.
o It has other benefits such as flexible budgets, focused operations, lower costs, and more disciplined
execution.
• Disadvantages of Zero-based Budgeting:
o It is time-consuming. It is a very time-intensive exercise to do every year against incremental
budgeting which is a far easier method.
o It is also very resource-intensive in nature. To make the entire budget from scratch additional numbers
of employees are needed.
• Outcome Budgeting:
• It is the practice of developing budgets based on the relationship between funding and expected results. It
enhances visibility into how government policies translate into spending and focus on the outcomes of a
funded activity i.e. the quality or effectiveness of services provided. It aims to align programs and services
with prioritized government outcomes.
• Incremental Budgeting: An incremental budget is a budget prepared using a previous period's budget or
actual performance as a basis with incremental amounts added for the new budget period.

Q 53.A
• India plans to ring in its era of space-to-space tracking and communication of its space assets this year by
putting up a new satellite series called the Indian Data Relay Satellite System.
• The IDRSS is a set of satellites to track, send and receive information from other Indian satellites. It is
planned to track and be constantly in touch with Indian satellites, in particular, those in low-earth orbits
that have limited coverage of earth. Hence, statement 1 is correct.
• In the coming years, it will be vital to Indian Space Research Organisation (ISRO), whose roadmap is
dotted with advanced LEO missions such as space docking, space station, as well as distant expeditions to
the moon, Mars and Venus. It will also be useful in monitoring launches, according to K. Sivan, ISRO
Chairman and Secretary, Department of Space.
• The first beneficiary would be the prospective crew members of the Gaganyaan mission of 2022 who can
be fully and continuously in touch with mission control throughout their travel.
• Work on the two IDRSS satellites planned initially has begun. The first of them will be sent towards the
end of 2020. It will precede the pre-Gaganyaan experimental unmanned space flight which will have a
humanoid dummy. A second one will follow in 2021. The two will offer near-total tracking, sending and
receiving of information from the crew 24/7.
• Older space majors such as the U.S. and Russia started their relay satellite systems in the late 1970s-80s
and a few already have around 10 satellites each. They have used them to monitor their respective space

18 www.visionias.in ©Vision IAS


stations Mir and the International Space Station and trips that dock with them, as well as the Hubble
Space Telescope.
• Statement 2 is not correct: IDRSS satellites of the 2,000 kg class would be launched on the GSLV
launcher to geostationary orbits around 36,000 km away. In such fixed orbits, they would be covering the
same area on earth. A satellite in GEO covers a third of the earth below and three of them can provide
total coverage.

Q 54.B
• The Balance of Payments can be divided under the following components -:
o Autonomous
▪ International economic transactions are called autonomous when transactions are made
independently of the state of the BoP (for instance due to profit motive).
▪ These items are called ‘above the line’ items in the BoP. The balance of payments is said to be in
surplus (deficit) if autonomous receipts are greater (less) than autonomous payments.
o Accommodating transactions
▪ These are termed ‘below the line’ items. These are determined by the net consequences of the
autonomous items, that is, whether the BoP is in surplus or deficit. Hence, statement 1 is not
correct.
▪ The official reserve transactions are seen as the accommodating item in the BoP (all others being
autonomous). Hence, statement 2 is correct.
o Errors and Omissions
▪ These constitute the third element in the BoP (apart from the current and capital accounts) which
is the ‘balancing item’ reflecting our inability to record all international transactions accurately
Q 55.C
• In January 2020, the South Australian Department for Environment and Water began a five-day
cull of the camels, the first mass cull of camels in the area. More than 5,000 wild camels were shot
dead in an aerial operation in the Anangu Pitjantjatjara Yankunytjatjara (APY) area governed by
Aboriginal Australians, in the remote north-west of South Australia. Struggling to survive a drought in the
region, the camels have caused problems after heading from the "arid landscape" towards areas populated
by humans in search of water. "Extremely large groups" of camels had been gathering in and around
communities. The safety of the young children was in danger and the culling operation was a last-resort
measure to try and control the feral pests following the highest standards of animal welfare.
• Australian feral camels are feral populations consisting of two species of camel: mostly dromedaries
(Camelus dromedarius) but also some Bactrian camels. Imported into Australia from British India and
Afghanistan during the 19th century for transport and construction during the colonization of the central
and western parts of Australia, many were released into the wild after motorized transport replaced the use
of camels in the early 20th century, resulting in a fast-growing feral population.

Q 56.A
• The economic cycle is the upward and downward movement of levels of GDP (gross domestic product)
and refers to periods of expansion and contraction in the level of economic activities (business
fluctuations) around a long-term growth trend.
• The four stages of the economic cycle are also referred to as the business cycle. These four stages
are expansion, peak, contraction, and trough.
• During the expansion phase, the economy experiences relatively rapid growth, interest rates tend to be
low, production increases, and inflationary pressures build. The peak of a cycle is reached when
growth hits its maximum rate. Peak growth typically creates some imbalances in the economy that need
to be corrected. This correction occurs through a period of contraction when growth slows,
employment falls, and prices stagnate. The trough of the cycle is reached when the economy hits a low
point and growth begins to recover.
• Expansion:
• In the expansion phase, there is an increase in various economic factors, such as production,
employment, output, wages, profits, demand and supply of products, and sales. Also, in the
expansion phase, the prices of the factor of production and output increase simultaneously. In the
expansion phase, due to an increase in investment opportunities, idle funds of organizations or individuals
are utilized for various investment purposes.
19 www.visionias.in ©Vision IAS
Q 57.C
• Frictional unemployment is a type of unemployment that arises when workers are searching for new jobs
or are transitioning from one job to another. It is part of natural unemployment and hence is present even
when the economy is considered at full employment. Hence statement 1 is correct.
• The major cause of frictional unemployment include -:
o A mismatch between the workers and available jobs
▪ If there is a mismatch between job-seekers and available jobs in the market, that is considered
frictional unemployment. The issue can especially affect the new entrants or re-entrants to the job
market. This is generally due to the natural career progression for an employee, and their natural
transition to a new job, industry, or role.
o Workers’ dissatisfaction with work conditions
▪ Workers’ anxiety towards salaries, benefits, work location, job responsibilities, etc. may force
them to quit their current job, and look for something that better meets their updated expectations.
• In a recession, frictional unemployment tends to fall as workers have less confidence to wait for a better
job. They are more likely to stay in their current job or take the first job that comes along. Hence
statement 2 is correct.

Q 58.C
• Transfer payments are one-way payments of money for which no money, good, or service is received in
exchange. Governments use such payments as means of income redistribution by giving out money under
social welfare programs such as social security, old age or disability pensions, student grants,
unemployment compensation, etc.
• On the other hand, Subsidy is the benefits given to someone as a tax rebate or money for some economic
purpose like keeping the price stable, promotion of exports, etc. These are counted as expenditure in
GDP. Hence subsidies are not counted as transfer payments.
• Hence option (c) is the correct answer.

Q 59.C
• Gross Domestic Product (GDP) is an economic measure of total income and output for a given period
(usually a year) within the domestic territory of a nation. Economists use GDP to measure the relative
wealth and prosperity of different nations, as well as to measure the overall growth or decline of a nation's
economy. GDP includes all private and public consumption, government outlays, investments,
additions to private inventories, paid-in construction costs, and the foreign balance of trade (exports
are added, imports are subtracted).
• For economic calculations such as GDP, GNP, GVA etc. the concept of domestic/economic territory is
used in place of political or geographical territory.
• The economic territory of India includes the following:
o The area lying within the political borders, including the territorial waters of the nation.
o Ships and aircraft, run by the countrymen, between two or more than two countries.
o Floating platforms, fishing vessels, oil, and natural gas rigs operated by the nationals, in the
international waters or involved in the process of extraction, in the region, where the country has
exclusive rights.
o Military establishments, embassies, and consulates of the country, located outside.
• Salaries of Indian employees working in Embassy of Japan in India will not be included in the Gross
Domestic Product of India as the embassy of Japan is not a part of the domestic territory of
India. Salary of residents of Japan working in the Indian embassy in China will be included in the
Gross Domestic Product of India as the Indian Embassy is a part of the domestic territory of India.
• Even though GDP is frequently used to capture the wellbeing of a society, it leaves out important aspects
of the economy such as
o GDP doesn’t measure some very useful output because it is unpaid (homemakers’ services,
parental child care, volunteer efforts, home improvement projects) called non-market
transactions.
o GDP does not include output from the Underground Economy. The underground economy refers
to economic transactions that are deemed illegal, either because the goods or services traded are
unlawful, or because transactions fail to comply with governmental reporting requirements. This is
not included in GDP calculation.

20 www.visionias.in ©Vision IAS


o Intermediate goods and services, which are used in the production of final goods and services, are
not included in the calculation of GDP because expenditures on intermediate goods and services are
included in the market value of expenditures made on final goods and services.
o GDP fails to account for the degree of income inequality in society.
o GDP fails to account for negative externalities ( the costs imposed on human health and the
environment ) arising from the production or consumption of the nation’s output.
• Hence option (c) is the correct answer.

Q 60.C
• The Reserve Bank of India has recently announced the external benchmark linked lending rates over the
existing marginal cost of funds-based lending rate (MCLR) system. According to it, banks must link all
new floating rate loans to an external benchmark like repo rate, 3-month or 6-month treasury bill yield, or
any other benchmark published by the Financial Benchmarks India Pvt. Ltd. Hence statement 1 is
correct.
• The new regime will only apply to banks and not NBFCs. Leading housing finance companies like HDFC
will not be under the purview of the new regime. However, if repo-linked rates of other lenders come
down, the competition will force NBFCs to match the rates. Hence statement 2 is correct.
• The repo (or repurchase) rate is the rate at which the Reserve Bank of India (RBI) lends money to other
banks. Hence, cuts in the repo rate are meant to lead to cuts in the home loan and other lending rates as
banks get to borrow money cheaply from the RBI. The pegging of rate to an external benchmark will
enable in the faster monetary transmission of rate cuts than has happened so far under the MCLR system.

Q 61.D
• The rupee is the standard unit of money in India, but the face value of coins or currency notes is
greater today than its real value. It does not have any intrinsic value and it is a type of token
money. Token money is made of cheaper metal; it is limited legal tender; it is not subject to free coinage
and its face value is greater than its intrinsic or metallic value. Token money consists of small
coins. Hence statement 1 is not correct.
• Under the Minimum Reserve System, the RBI has to keep a minimum reserve of Rs 200
crore comprising of gold coin and gold bullion and foreign currencies. Under the Minimum Reserve
System, the RBI can issue an unlimited amount of currency by keeping the reserve. The minimum reserve
is a token of confidence and does not have any practical connection with the amount of new currencies
issued by the RBI. Under the Minimum Reserve System, RBI can issue an unlimited amount of currency
by keeping the reserve (without backing the total currency in circulation with the gold of equal
value). But RBI follows some principle or rule for issuing new currencies based upon the economic
growth and transaction needs of the people. Hence statement 2 is not correct.
• According to RBI rules, those who refuse to accept the Indian currency can be liable for action under
section 124A (sedition) of IPC. On September 20, RBI clarified the coin was valid and anyone refusing it
was liable to face legal action. Hence statement 3 is correct.

Q 62.C
• The National Strategy for Financial Inclusion 2019-2024 sets forth the vision and key objectives of the
financial inclusion policies in India to help expand and sustain the financial inclusion process at the
national level through a broad convergence of action involving all the stakeholders in the financial sector.
• The strategy aims to provide access to formal financial services affordably, broadening & deepening
financial inclusion and promoting financial literacy & consumer protection.
• The National Strategy for Financial Inclusion for India 2019-2024 has been prepared by RBI under
the aegis of the Financial Inclusion Advisory Committee and is based on the inputs and suggestions from
Government of India, other Financial Sector Regulators viz., Securities Exchange Board of India (SEBI),
Insurance Regulatory and Development Authority of India (IRDAI) and Pension Fund Regulatory and
Development Authority of India (PFRDA).

Q 63.C
• Statement 1 is correct: BHIM (Bharat Interface for Money) is a payment interface developed by the
National Payments Corporation of India (NPCI) to allow real-time fund transfer. It is an app that lets you
make simple, easy and quick payment transactions using Unified Payments Interface (UPI). One can make

21 www.visionias.in ©Vision IAS


instant bank-to-bank payments and Pay and collect money using just a Mobile number or Virtual Payment
Address (VPA).
• Statement 2 is correct: Recently, the government has launched the new version of BHIM referred to as
BHIM 2.0. to make it more feature-rich and effective. Some of the striking features marking BHIM 2.0
include a 'Donation' gateway, increased transaction limits for high-value transactions, linking multiple
bank accounts, offers from merchants, the option of applying in IPO, gifting money. Also, the support
for three languages namely Konkani, Bhojpuri and Haryanvi has been added.

Q 64.B
• Market prices are the prices as paid by the consumers Market prices also include product taxes and
subsidies. The term factor cost refers to the prices of products as received by the producers. Thus, factor
cost is equal to market prices, minus net indirect taxes.
• Net indirect taxes are calculated by subtracting subsidies from indirect taxes.
• Factor Cost = Market Price-Net Indirect taxes = Market Price - (Indirect taxes-subsidies)
• The market price is greater than factor cost when the net indirect tax is positive, that is, indirect taxes are
more than subsidies and market prices are less than the factor cost when the net indirect tax is
negative, that is, indirect taxes are less than the subsidies. Hence option (b) is correct.
• An increase or decrease in the cost of production will increase or decrease the factor cost as well as
market price in a proportionate amount.

Q 65.C
• The Forest Advisory Committee (FAC) of the environment ministry has recently approved a
scheme that could allow “forests” to be traded as a commodity as it would allow the Forest
Department to outsource its responsibility of reforesting to non-government agencies. The FAC is an apex
body tasked with adjudicating requests by the industry to raze forest land for commercial use.
• Currently, industries need to find appropriate non-forest land in place of and equal to the forest land that
was razed. The industry would also have to pay the State Forest Department the current economic
equivalent of the forest land. It is then the responsibility of the Forest Department to restore vegetation in
that area, which would over time grow into forests. The new scheme approved by the Forest Advisory
Committee called the ‘Green Credit Scheme’ would allow agencies like private companies and
village forest communities to identify land and grow plantations.
• After three years, they would be eligible to be considered as compensatory forest land if they met the
criteria set by the Forest Department. An industry in need of forest land could then pay for these patches
of forest land, and this would then be transferred to the Forest Department. Previously, in 2015, a ‘Green
Credit Scheme’ for degraded forest land with public-private participation had been recommended but was
shelved when it was not approved by the Union environment ministry.

Q 66.B
• The government has created a Department of Military Affairs in the Defence Ministry and it will
be headed by newly-appointed Chief of Defence Staff (CDS). Hence, only statement 2 is correct.
• The new department will have under it works relating to the three forces -- Army, Navy and Air Force --
and procurement exclusive to the services, except capital acquisitions, according to prevalent rules and
procedures.
• The Department of Military Affairs will have an integrated headquarters of the Defence Ministry
comprising of the Army headquarters, the Naval headquarters, the Air headquarters, the Defence Staff
headquarters, and the Territorial Army.

Q 67.A
• Because of linkages being developed in recent times between economic offenses and threats to national
security, the Government constituted Economic Intelligence Council (EIC) under the chairmanship of
the Finance Minister in the year 1990 to facilitate coordination amongst the enforcement agencies
dealing with economic offenses. The Council is also the apex of 30 regional economic intelligence
committees and is part of the Union Ministry of Finance. Hence statements 1 and 2 are not correct.
• The mandate of the EIC is to combat generation and laundering of Black Money through intelligence
gathering on issues relating to economic security, provide inter-agency coordination and deal with
economic offenses effectively. Hence statement 3 is correct.

22 www.visionias.in ©Vision IAS


• Hence option a is the correct answer.
• Because of the newer forms of continuously emerging complex economic crimes, the Govt. has put in
place institutional structure and mechanism for coordinated intelligence sharing and investigation at the
regional level in the form of 30 Regional Economic Intelligence Councils (REICs).
• The REIC forum consists of officers from CBDT, CBIC, heads of other related agencies both from
Central & State governments. REICs have to gather economic intelligence on trade & industry in region
and examine trends and modus operandi and suggest measures for dealing effectively against economic
offenders.

Q 68.C
• It has now become mandatory for all jewelers to sell hallmark gold jewelry. Hallmarking is the accurate
determination and official recording of the proportionate content of the precious metal. Gold hallmarking
is a purity certification of the precious metal. In India, at present gold and silver have been brought under
the purview of hallmarking. Hallmarking will be done for jewelry in 3 categories - 14 carats, 18 carats and
22 carats.
• Hallmark on gold jewelry now has four marks:
o BIS Mark
o Purity in carat and Fineness (it can be any of the following)
▪ 22K916 Corresponding to 22 Carat
▪ 18K750 Corresponding to 18 Carat
▪ 14K585 Corresponding to 14 Carat
▪ Assaying/Hallmarking Centre’s Identification Mark/Number
▪ Jeweller’s Identification Mark/number
• Before 01 January 2017, Gold Hallmark also consisted of Year of marking – denoted by a code letter
e.g. ‘A’ for year 2000 or ‘B’ for year 2001 however, that has now been discontinued. The year of making
the jewellery is not a part of the BIS hallmark.

Q 69.C
• e-Kuber is the Core Banking Solution of the Reserve Bank of India. It provides the provision of a
single current account for each bank across the country, with decentralized access to this account from
anywhere-anytime safely using portal-based services. Hence option (c) is the correct answer.
• It enables ease of operations. The system also benefits state /central Governments as users. Some of the
facilities offered include the provision of portal-based access which allows Government departments to
access on the anywhere-anytime basis and view their balances – of all types including the Ways and
Means Advances, drawings, funds positions and the like – all in a consolidated manner to help them in
better funds management.
• The capability of consolidating revenue collections by banks through the e-Kuber offers the potential for
better flexibility for the Government in managing its finances apart from moving over towards higher
levels of electronic banking.

Q 70.D
• Statement 1 is not correct: The printing and circulation of currency notes are regulated by the Reserve
Bank of India Act, 1934. As per the act, the Reserve Bank manages currency in India. The responsibility
for coinage vests with the Government of India based on the Coinage Act, 2011 as amended from time to
time.
• Statement 2 is not correct: The Reserve Bank of India is responsible for printing the currency notes as
per the Reserve Bank of India Act, 1934. However, the denomination of the currency is decided by
the Central Government. The Government, on the advice of the Reserve Bank, decides on various
denominations of banknotes to be issued. The Reserve Bank can also issue banknotes in the
denominations of five thousand rupees and ten thousand rupees or any other denomination that the Central
Government may specify. However, there cannot be banknotes in denominations higher than ten thousand
rupees in terms of the current provisions of the Reserve Bank of India Act, 1934.

23 www.visionias.in ©Vision IAS


Q 71.C
• Broad money = Narrow money + time deposits held by commercial banks and post office savings
organization.
• At any point in time, the money held with the public has two most liquid components:
o Currency Component: This consists of all the coins and notes in circulation.
o Demand Deposit Component: Demand Deposit component is the money of the general public with the
banks, which can be withdrawn by them using cheques, withdrawals, ATMs, etc. The above two
components are of Narrow Money. It is denoted by RBI as M1. It is the most liquid part of the money
supply because the demand deposits can be withdrawn anytime during the banking hours.
• Time deposits have a fixed maturity period and hence cannot be withdrawn before the expiry of this
period. M3 or Broad Money is therefore calculated as Narrow Money + Time Deposits of the public with
banks. Hence the correct answer is option (c).
• Broad money does not include the interbank deposits of the public with all banks including the
cooperative banks.
• Cash Reserves of the Banks held with RBI are counted under M0 i.e. The monetary base.

Q 72.C
• The Reserve Bank of India gives temporary loan facilities to the centre and state governments as a
banker to the government. This temporary loan facility is called Ways and Means Advances (WMA).
Hence option c is the correct answer.
• This facility is provided to help them tide over temporary mismatches in the cash flow of their receipts
and payments. This is guided under Section 17(5) of the RBI Act, 1934.
• There are two types of WMA for states– normal and special. While Normal WMA are clean advances
without collateral, Special WMA are secured advances provided against the pledge of the government of
India–dated securities.
• The WMA scheme for the Central Government was introduced on April 1, 1997. The WMA scheme was
designed to meet temporary mismatches in the receipts and payments of the government. This facility can
be availed by the government if it needs immediate cash from the RBI. The WMA is a loan facility form
the RBI for 90 days which implies that the government has to vacate the facility after 90 days. The
interest rate for WMA is currently charged at the repo rate. The limits for WMA are mutually decided by
the RBI and the Government of India.

Q 73.B
• The government has recently launched a new regular employment-unemployment survey, namely,
Periodic Labour Force Survey (PLFS), 2017 with changes in survey methodology, data collection
mechanism, and sampling design vis-à-vis the earlier quinquennial (once in every five years) Employment
and Unemployment surveys of NSSO. It is the first annual PLFS, 2017 which provides
quarterly employment statistics for urban areas on Current Weekly Status (CWS) basis and annual
estimates of employment indicators both in rural and urban areas on CWS and usual status basis. Hence
statement 1 is correct.
• The PLFS has been launched with the objective of measuring quarterly changes of various labor market
statistical indicators in urban areas as well as generating annual estimates of these indicators both for rural
and urban areas, which can be used for policymaking.
• As per the report, the unemployment rate is estimated at 6.1% at the all-India level as per the usual status
criterion. Further, between the sexes, the unemployment rate is higher among the urban females at 10.8%
than their male counterparts, estimated at 7.1%. In the rural areas, the male unemployment rate, estimated
at 5.8%, exceeded the female rate at 3.8%. Hence statement 2 is correct.
• As per the report, Nagaland has the highest unemployment rate among the states at 21.4%, and Meghalaya
the lowest at 1.5%. Also, the survey shows huge variations among the states. Besides Nagaland, states
with high unemployment are Goa and Manipur. Chhattisgarh and Sikkim are among those with the lowest
rates. Hence statement 3 is not correct.

24 www.visionias.in ©Vision IAS


Q 74.A
• Recently, a Committee has been constituted to work out the modalities and prepare the guidelines for
pooling resources, manpower and technology for the merger of Rajya Sabha and Lok Sabha Television.
The Committee is expected to submit its report to Rajya Sabha Chairman and Lok Sabha Speaker. The
final decision will be taken by them.
• Prasar Bharati is a statutory autonomous body established under the Prasar Bharati Act and came into
existence on 23.11.1997. It is the Public Service Broadcaster of the country. The objectives of public
service broadcasting are achieved in terms of the Prasar Bharati Act through All India Radio and
Doordarshan.
o Doordarshan is India's Public Service Television network, the TV vertical of Prasar Bharati. DD
News is the only terrestrial cum satellite News Channel of the country.
o All India Radio is the Radio vertical of Prasar Bharati. Its motto is – ‘Bahujan Hitaya: Bahujan
Sukhaya’. It is one of the largest broadcasting organisations in the world in terms of the number of
languages of broadcast and the spectrum of socio-economic and cultural diversity it
serves. Terrestrially, AIR originates programming in 23 languages and 179 dialects.
o Free Dish is the Free Direct to Home (DTH) service of Prasar Bharati for receiving TV service
directly through satellite with a personal small dish antenna. DTH service does not require a local
cable operator for receiving TV service at home.
o Prasar Bharati is also introducing Digital Terrestrial transmission for the first time in India.
• Lok Sabha TV is the first parliamentary channel in India. It is dedicated to the Lower House of the
Parliament. Owned and operated by Lok Sabha Secretariat, LSTV has the mandate to telecast
uninterrupted live proceedings of the Lok Sabha. Committed towards its role as a Public
Broadcaster. Rajya Sabha TV is owned and operated by the upper house of the Indian Parliament.

Q 75.D
• The maximum residue limit (also maximum residue level, MRL), is the maximum amount of pesticide
residue that is expected to remain on food products when a pesticide is used according to label directions,
which will not be a concern to human health.
• The Codex Committee on pesticide residues (CCPR) develops and maintains acceptable pesticide
Codex (maximum residue limit) MRLs for food commodities in international trade. Although these
form the basis of globally accepted standards, an increasing number of developing countries and
economies in transition set their own independent standards.
• Hence option (d) is the correct answer.

Q 76.C
• Statement 1 is correct: Annual
Status of Education Report (ASER),
India’s largest NGO-run annual
survey, has been conducted by
Pratham since 2005 to provides
estimates of children’s schooling
status and their ability to read
simple text and do basic arithmetic
and evaluate the relevance and
impact of its programs. Findings are
disseminated at national, state, district
and village levels, and influence
education policies at both state and
central levels.
• Statement 2 is correct: ASER
Survey is a household survey. It
consists of one-on-one oral
assessments. It is aimed at the
representative sample of all children (whether in school or out of school). It focuses on foundational skills
like reading and maths. It is limited to rural areas of the country. It is a citizen-led survey.

25 www.visionias.in ©Vision IAS


• Recently, the Annual Status of Education Report (ASER)'s 'Early Years' report 2019 was released
by the NGO Pratham. The survey was conducted in 26 districts across 24 states in India covering more
than 36000 children in the age group 4 to 8.
• It highlighted that overall, more than 90% of young children in the age group 4-8 are enrolled in some
type of educational institution. Gender gaps are visible even among these young children, with more girls
than boys enrolled in government educational institutions and more boys than girls enrolled in private
institutions. Among four-five-year-old children, 56.8% of girls and 50.4% of boys were enrolled in
government schools or preschools, whereas 43.2% of girls and 49.6% of boys were enrolled in private
preschools or schools, the survey found.
• At least 25% of school children in the four-eight age group do not have age-appropriate cognitive and
numeracy skills, making for a massive learning deficit at a very early stage, according to the Annual
Status of Education Report (ASER).
• The government preschool system is managed through the Centre’s Integrated Child Development
scheme, under the ministry of women and child development, while schools come under the
education ministries at the Centre and in the states.

Q 77.A
• Veblen Goods: Goods for which an increase in price encourages people to buy more of it. This is because
they think more expensive goods are better.
• A Veblen good, like a Giffen good, has an upward-sloping demand curve, which runs counter to the
typical downward-sloping curve. However, a Veblen good is generally a high-quality, coveted product, in
contrast to a Giffen good which is an inferior product that does not have easily available substitutes. The
increase in demand for a Veblen good reflects consumer tastes and preferences, unlike a Giffen good,
where higher demand is directly attributable to the price increase.
• Very expensive products – such as designer jewellery, pricey watches, and luxury cars – that are marketed
as being 'exclusive', or which convey the appearance of success, can be classified as Veblen goods.
• Veblen goods contradict the basic law of demand – which states that quantity demanded has an inverse
relationship with price.

Q 78.D
• The Securities and Exchange Board of India was officially appointed as the authority for regulating the
financial markets in India on 12th April 1988. It was initially established as a non-statutory body but later
in 1992, it was declared an autonomous body with statutory powers under the SEBI Act, 1992. SEBI plays
an important role in regulating the securities market of India.
• Following the recommendations of the Malhotra Committee report, in 1999, the Insurance
Regulatory and Development Authority (IRDA) was constituted as an autonomous body to regulate
and develop the insurance industry. The IRDA was incorporated as a statutory body in April 2000
under the Insurance Regulatory and Development Authority (IRDA) Act, 1999. The key objectives
of the IRDA include the promotion of competition to enhance customer satisfaction through increased
consumer choice and lower premiums while ensuring the financial security of the insurance market.
• National Bank for Agriculture and Rural Development (NABARD) was established on 12 July 1982
by an Act of the Parliament under the NABARD Act, 1981. As a Development Bank, it is mandated to
provide for and regulate credit and other facilities for the promotion and development of agriculture, small
scale industries, cottage, and village industries, handicrafts and other rural crafts and other allied
economic activities in rural areas to promote integrated rural development and securing prosperity of rural
areas, and for matters connected.
• As per the recommendations of the Working Group on Rural Banks (Narsimhan Committee), the
regional rural banks were established in 1975 for supplementing the commercial banks and co-
operatives in supplying rural credit. The main objective of regional rural banks in India is to advance
credit and other facilities, especially to small and marginal farmers, agricultural laborers, artisans and
small entrepreneurs to develop agriculture, trade, commerce, industry and other usual productive activities
in different rural areas of the country.

26 www.visionias.in ©Vision IAS


Q 79.B
• Statement 1 is not correct: Open Market Sales Scheme refers to the selling of foodgrains by
Government/Government agencies at predetermined prices in the open market from time to time to
enhance the supply of grains especially during the lean season and thereby to moderate the general open
market prices especially in the deficit regions.
• Recently, with food grain stocks held by state-owned Food Corporation of India (FCI) in the central pool
hit a six-year high, the government increased its allocations to below-poverty-line (BPL) families and
raised the open market sale scheme (OMSS) to liquidate inventory.
• Statement 2 is correct: Under the OMSS, FCI releases wheat and rice at predetermined prices in the
open market from time to time to enhance the supply of wheat and rice especially during the lean season
to moderate the open market prices.
• The present form of OMSS(Domestic) comprises of 3 schemes as under:
o Sale of wheat to bulk consumers/private traders through e-auction.
o Sale of wheat to bulk consumers/private traders through e-auction by dedicated movement.
o Sale of Raw Rice Grade ‘A’ to bulk consumers/private traders through e-auction.

Q 80.C
• National Disposable Income is the maximum amount of goods and services a country has which
could be used for the purpose of saving and consumption.
• It is obtained by adding Net National Product at market prices with current transfers from the rest of the
world. It gives an idea of what is the maximum amount of goods and services the domestic economy has
its disposal. This is the maximum amount of goods and services a country has which could be used for the
purpose of saving and consumption.
• Hence option (c) is the correct answer.

Q 81.B
• Tax revenue is charged on income earned by an individual or an entity (direct tax) and on the value of the
transaction of goods and services (indirect tax). On the other hand, non-tax revenue is charged against
services provided by the government. It also includes interest charged on loans advanced by the
government for various purposes
• Non-tax revenue includes-
o Interest: It comprises of interest of loans given to states and union territories for reasons like non-plan
schemes (e.g. flood control) and planned schemes with maturity period of 20 years such as
modernization of police forces and also interest on loans advanced to Public Sector Enterprises
(PSEs), Port Trusts and other statutory bodies, etc.
o Dividends and profits: This includes dividends and profits from PSEs as well as the transfer of
surplus from Reserve Bank of India (RBI).Petroleum license: This includes fees to get the exclusive
right for exploration in a particular region. Such fees may be in the form of royalty, share of the profit
earned from contact areas during a specific period, Petroleum Exploration License (PEL) fee or
Production Level Payment (PLP).
o Power supply fees: This includes fees received by the Central Electricity Authority from the supply
of power under the Electricity (Supply) Act.
o Fees for Communication Services: This mainly includes the license fees from telecom operators
on account of spectrum usage charges that licensed Telecom Service Providers to pay to the
Department of Telecom (DoT).
o Broadcasting fees: It includes license fees paid by DTH operators, commercial TV services,
commercial FM radio services, etc.
o Road, Bridges usage fees: This includes receipts through toll plazas on account of the usage of
national highways, permanent bridges, etc.
o Examination fees: This includes fees paid by applicants of competitive examinations conducted by
the Union Public Service Commission (UPSC) and Staff Selection Commission (SSC) to fill up
vacancies in government offices.
o Fee for police services: This includes fee received for supplying central police forces to state
governments and other parties like Central Industrial Police Force (CISF) to industries etc.
o Sale of stationery, gazettes, etc: This includes receipts under ‘Stationery and Printing’ relating to the
sale of stationery, gazettes, government publications, etc.

27 www.visionias.in ©Vision IAS


o Fee for Administrative Services: This includes fees received for providing services like audit
services, issuance of passport, visa, etc.
o Receipts relating to Defence Services: This relates to services provided through Canteen Stores
Department (CSD).
• Disinvestment receipts form a part of the Non-debt capital receipts.

Q 82.D
• As per the estimates from Central Statistics Office (CSO), the share of agriculture and allied sector in
GDP of the country has declined from 47.6% in 1960-61 to 16.9% in 2018-19 at current prices.
GDP share of Agriculture
Year
and Allied Sectors
1960-61 47
1990-91 29.5
2000-01 22.3
2018-19 16.1
• From the above table, it can be said that share of agriculture in GDP has declined after 1991 economic
reforms. Hence statement 2 is not correct.
• The services sector contributed 54.3% to India’s GVA in 2018-19. Share of employment of services
sector is at 34%, which is significantly lower than its share in GVA. Hence statement 1 is not correct.
• Hence option d is the correct answer.

Q 83.B
• In India, total Central Government Liabilities constitutes the following three categories:
o Internal Debt
o External Debt
o Public Account Liabilities
• However, Public Debt in India includes only Internal and External Debt incurred by the Central
Government. Hence statement 1 is not correct.
• The internal debt of the Central Government constituted 83.4 percent of public debt for 2019. Hence
statement 2 is correct.

Q 84.D
• Gross Valued Added (GVA) is a measure of total output and income in the economy. It provides the
rupee value for the number of goods and services produced in an economy after deducting the cost of
inputs and raw materials that have gone into the production of those goods and services. GDP is the sum
of private consumption, gross business investments in the economy, government investment,
government spending and net foreign trade (the difference between exports and imports). GVA at
basic prices is the sum of a country’s GDP and net of subsidies and taxes in the economy. While GVA

28 www.visionias.in ©Vision IAS


gives a picture of the state of economic activity from the producers’ side or supply side, the GDP
gives the picture from the consumers’ side or demand perspective. Hence statement 1 is not correct.
• A sector-wise breakdown provided by the GVA measure helps policymakers decide which sectors need
incentives or stimulus and accordingly formulate sector-specific policies. But GDP is a key measure when
it comes to making cross-country analyses and comparing the incomes of different economies.
• The government had started analyzing growth estimates using GVA methodology from January 2015.
Central Statistical Office (CSO) has started using GDP as the main measure of economic activities since
January 15, 2018. RBI switched back to the gross domestic product (GDP) based measure to offer its
growth estimates from the gross value added (GVA) methodology, citing global best practices. Hence
statement 2 is not correct.

Q 85.D
• Tax Expenditures refers to the opportunity cost of taxing at concessional rates, or the opportunity cost
of giving exemptions, deductions, rebates, deferrals credits, etc. to the taxpayers. Tax expenditures
indicate how much more revenue could have been collected by the Government if not for such
measures. In other words, it shows the extent of indirect subsidy enjoyed by the taxpayers in the
country. So, giving tax incentives will increase the tax expenditure by the government.
• Revenue receipts refer to those receipts which neither create any liability nor cause any reduction in
the assets of the government. They are regular and recurring in nature and the government receives them
in its normal course of activities. Revenue receipts of the government are generally classified under two
heads
o Tax Revenue: Tax revenue refers to the sum total of receipts from taxes and other duties imposed by
the government. Tax is a compulsory payment made by people and companies to the government
without reference to any direct benefit in return. Due to tax incentives, tax revenue will decrease.
As a result revenue receipts will decrease.
o Non-Tax Revenue: It includes interest received on loans, profits and dividends, fees and penalties,
etc.
• Hence option d is the correct answer.

Q 86.C
• The natural rate of interest is also called the neutral interest rate is the theoretical short-term interest rate
that would support the economy at the maximum output or full-employment GDP while
keeping inflation stable.
• It is the rate of interest on loans which is neutral in respect to commodity prices and tends neither to raise
nor to lower them
• The neutral rate is often referred to by central banks when making decisions about the bank rate since this
neutral rate is essentially the dividing line between expansionary and contractionary monetary
policy. Hence option (c) is the correct answer.

Q 87.A
• Statement 1 is not correct: E-cigarettes/E-Vaping are the most common form of Electronic Nicotine
Delivery Systems (ENDS). These are devices that do not burn or use tobacco leaves. Instead, they
vaporize a solution using a battery. This vapor is then inhaled by the user. E-cigarettes are electronic
devices that can enable the delivery of all intoxicating substances. Predominantly, they are used for
nicotine delivery, which is one of the most addictive elements known. This also includes all forms of
electronic nicotine as well as non-nicotine delivery devices such as e-hookahs and heat-not-burn
products. An electronic cigarette is a battery-operated device that emits doses of vaporized nicotine, or
non-nicotine solutions, for the user to inhale.
• Statement 2 is correct: E-Vaping aims to provide a similar sensation to inhaling tobacco smoke, without
the smoke. Smoke is the result of combustion. When combustion occurs, new chemicals form through
the process of oxidation. Smoke contains thousands of new chemicals different from those initially
burned. Fire is what creates smoke. When you smoke a cigarette, a fire must be involved to create the
smoke. Vapors: When a substance becomes gaseous, at a temperature that is lower than its point of
combustion, it is considered vapor. The chemicals that are in vapor are the same as those found in the
vaporized substance.

29 www.visionias.in ©Vision IAS


• Statement 3 is correct: Paving the way for a complete ban on e-cigarettes, the Rajya Sabha passed
the Prohibition of Electronic Cigarettes (Production, Manufacture, Import, Export, Transport, Sale,
Distribution, Storage and Advertisement) Bill, 2019, by voice vote.
The Bill has already been passed by the Lok Sabha for replacing the ordinance promulgated last
September.

Q 88.B
• Recently, an employment tribunal in the United Kingdom determined that ethical veganism meets the test
required to be a philosophical belief, because of which it is protected under the Equality Act, 2010. The
Equality Act, 2010, protects people from discrimination in the workplace and the wider society in the UK.
Under the Act, a belief is defined as any religious or philosophical belief. Since the tribunal has ruled that
ethical veganism is a philosophical belief, it is a protected characteristic under the Act.
• A vegan is someone who does not eat or use animal products. Some people choose to simply follow a
vegan diet - that is, a plant-based diet avoiding all animal products such as dairy, eggs, honey, meat, and
fish. But ethical vegans try to exclude all forms of animal exploitation from their lifestyle. For instance,
they avoid wearing or buying clothing made from wool or leather, or toiletries from companies that carry
out animal testing.
• An ethical vegan is someone whose lifestyle and choices are shaped by their desire to avoid cruelty
and suffering to animals at all practical costs. Ethical veganism goes far beyond a plant-based
diet. The limits an ethical vegan faces don’t just stop at their food choices.

Q 89.A
• Fiscal slippage in simple terms is any deviation in expenditure from the expected. When the
Government's expenditure surpasses the expected or estimated levels, the nation might face the threat
of fiscal slippage. So Government actions that increase the expenditure will lead to the condition of fiscal
slippage.
• Examples include
o Income support schemes,
o UDAY scheme (revival package for power distribution companies) and
o farm loan waivers as such schemes lead to extra government expenditure.
o Tax rate cuts leading to a decline in tax revenue
• Disinvestment proceeds increase the capital receipts of the Government. So, it will have the effect of
the prevention of the situation of fiscal slippage.
• Hence option a is the correct answer.

Q 90.C
• The Pradhan Mantri Rojgar Protsahan Yojana (PMRPY) scheme has been designed to incentivize
employers for the generation of new employment. It is a scheme where the Government of India pays
the full employer's contribution towards Employees' Provident Fund (EPF) scheme and the Employees'
Pension Scheme (EPS), for the new employees, for the first three years of their employment. The scheme
is being implemented by the Ministry of Labour and Employment. Hence statement 1 is not correct and
statement 3 is correct.
• All establishments registered with Employees' Provident Fund Organisation (EPFO) under EPF Act 1952
can apply for availing benefits under the scheme. They need to have a valid Labour Identification Number
(LIN) allotted to them under the Shram Suvidha Portal. Hence statement 2 is correct.
• This scheme has a dual benefit, where, on the one hand, the employer is incentivized for increasing the
employment base of workers in the establishment, and on the other hand, a large number of workers will
find jobs in such establishments. A direct benefit is that these workers will have access to the social
security benefits of the organized sector.
Q 91.B
• Pongal is an occasion for the farming community to take a break and play some games after the harvest
and before the next sowing season. Pongal evenings are filled with much banter and laughter in the State
and playing games is an integral part of these celebrations.
• Villagers gather in a circle and a 10-feet long wooden pole would be plonked in the center, the surface of
which would be slathered with oil. Young men would be tasked with climbing up the pole. This game is
called vazhukku maram (slippery pole). Those who manage to reach the top win prize money.

30 www.visionias.in ©Vision IAS


• Mallar kambam, is a combination of gymnastics and yoga. It also involves climbing up a pole, but this
is not a slippery one. Similarly, there’s a game called kazhumaram played during Pongal in Dindigul,
Coimbatore, Thanjavur, and Theni districts. These regions grow coconuts and farmers there are used to
climbing trees. They exhibit their prowess through the game.
• In urban pockets, the most popular Pongal game is uri adithal, which involves players breaking a pot
that is suspended at a certain height, blindfolded. “It’s similar to dahi (curd) handi played during
Janmashtami in Maharashtra. Kabaddi matches and silambam performances are also equally popular.
Games such as elavatta kal are regaining popularity.

Q 92.B
• National Infrastructure Pipeline is the investment plan unveiled by the Central Government for
enhancing infrastructure in identified sectors for five years from 2020-25.
• Funding of the scheme: The funding of the National Infrastructure Pipeline will be jointly made by the
Centre, states and the private sector in the proportion of 39:39:22 (39 % each by the centre and states
and 22% by the private sector).
• The Task Force on National Infrastructure Pipeline that chaired by Secretary, Department of Economic
Affairs (DEA), Ministry of Finance (MoF) submitted a detailed report on the infrastructure plan. The
main function of the task force was to identify technically feasible and financially/ economically viable
infrastructure projects that can be initiated in fiscals 2020 to 2025.
• The Taskforce has proposed certain goals, strategies, and standards under its Infrastructure Vision 2025.
The following are the components of the vision.
o Affordable and clean energy
o Digital Services
o Quality Education
o Convenient and efficient transportation and logistics
o Housing and water supply for all
o Disaster-resilient standards-compliant public infrastructure
o Agriculture infrastructure
o Good health and well being

Q 93.C
• Consumer Price Index (CPI) is the index of prices of a given basket of commodities that are bought by the
representative consumer. The Gross Domestic Product (GDP) deflator is a measure of general price
inflation. It is calculated by dividing nominal GDP by real GDP and then multiplying by 100.
• Comparison between GDP Deflator and CPI:
o The goods purchased by consumers do not represent all the goods that are produced in a country
hence the basket of CPI is limited. On the other hand, GDP deflator takes into account all such goods
and services hence it's the basket is greater than CPI.
o CPI includes prices of goods consumed by the representative consumer, hence it includes prices of
imported goods. GDP deflator does not include the prices of imported goods. Hence statement 1 is
correct.
o The weights are constant in CPI – but they differ according to the production level of each good in
GDP deflator. Hence statement 2 is correct.

Q 94.B
• The waterfall mechanism under the Insolvency and Bankruptcy Code outlines the order of priority
for repayment to creditors in the event of a liquidation. Under this, secured creditors have to be paid
fully before any payments can be made to unsecured financial creditors who in turn have priority over
operational creditors.
• If any money is recovered after the dissolution of the CD (Corporate Debtors), the same may be
distributed as per waterfall in section 53 of the Insolvency and Bankruptcy Code (IBC).
• The proceeds from the sale of the liquidation assets shall be distributed in the following order of
priority and within the specified period and manner:
o the insolvency resolution process costs and the liquidation costs paid in full;
o the following debts which shall rank equally between and among the following:
▪ workmen’s dues for twenty-four months preceding the liquidation commencement date; and

31 www.visionias.in ©Vision IAS


▪ debts owed to a secured creditor in the event such secured creditor has relinquished security in the
manner set out in section 52;
o wages and any unpaid dues owed to employees other than workmen for twelve months preceding the
liquidation commencement date;
o financial debts owed to unsecured creditors;
o the following dues shall rank equally between and among the following:
▪ any amount due to the Central Government and the State Government including the amount to be
received on account of the Consolidated Fund of India and the Consolidated Fund of a State, if
any, in respect of the whole or any part of the period of two years preceding the liquidation
commencement date;
▪ debts owed to a secured creditor for any amount unpaid following the enforcement of security
interest;
o any remaining debts and dues;
o preference shareholders, if any; and
o equity shareholders or partners, as the case may be.
• Hence option (b) is the correct answer.

Q 95.A
• Effective Revenue deficit is a new term introduced in the Union Budget 2011-12. Revenue deficit is the
difference between revenue receipts and revenue expenditure. Before 2011, the accounting system
includes all grants from the Union Government to the state governments/Union territories/other bodies as
revenue expenditure, even if they are used to create assets. Such assets created by the sub-national
governments/bodies are owned by them and not by the Union Government. Nevertheless, they do result
in the creation of durable assets.
• According to the Finance Ministry, such revenue expenditures contribute to the growth in the economy
and therefore, should not be treated as unproductive. In the Union Budget (2011-12) a new
methodology has been introduced to capture the ‘effective revenue deficit’, which excludes
those revenue expenditures (or transfers) in the form of grants for the creation of capital assets.
Hence pair 1 is not correctly matched.
• Primary Deficit: Primary Deficit is the difference between the current year’s fiscal deficit (total income –
total expenditure of the government) and the interest paid on the borrowings of the previous year.
Normally, when the government raises a loan, it includes the interest amount. When that amount is
deducted from the principal loan amount, it is called a primary deficit. Primary Deficit indicates the
borrowing requirements of the government, excluding interest. Hence pair 2 is not correctly matched.
• Monetised Deficit: Monetized deficit is also known as the ‘net reserve bank credit to the government’.
It is that part of the government deficit which is financed solely by borrowing from the RBI. Since
borrowings from the RBI can be both short-term and long-term, therefore, the monetized deficit is the sum
of the net issuance of short-term treasury bills, dated securities (that is, long-term borrowing from the
RBI) and rupee coins held exclusively by the RBI, net of Government’s deposits with the RBI. Hence
pair 3 is correctly matched.
• Hence option (a) is the correct answer.

Q 96.A
• Gross Fixed Capital Formation (GFCF) measures the value of acquisitions of new or existing fixed
assets by the business sector, governments and "pure" households (excluding their unincorporated
enterprises) fewer disposals of fixed assets.
• GFCF is a component of the expenditure on the gross domestic product (GDP), and thus shows something
about how much of the new value-added in the economy is invested rather than consumed. One of the
reasons for low GFCF is due to low spending by the private sector in the capital asset. The growth of the
productive capacity of the economy of the State depends on its rate of capital accumulation and it is
assessed by estimating the Capital Formation of that State.
• The higher the rate of growth of Capital Formation, the higher would be the productive capacity of the
economy, whereas its paucity leads to the low level of production at a higher cost. Thus Capital Formation
serves as a very important indicator in measuring the magnitude of growth of productive potential of the
economy. Hence statement 1 is correct.

32 www.visionias.in ©Vision IAS


• Gross Fixed Capital Formation (as a percentage of GDP) in the last 5 years as provided in
Economic Survey 2018-19.
Public sector Private sector
year Total GFCF
GFCF GFCF
2013-14 7.1 24.2 31.3
2014-15 7.0 23.1 30.1
2015-16 7.5 21.3 28.7
2016-17 6.8 21.4 28.2
2017-18 7.2 21.5 28.6
• From the above table it is clear that GFCF in India has not steadily increased in the last 5 years.
Hence statement 2 is not correct.

Q 97.A
• A sinking fund is a fund established by an economic entity by setting aside revenue over some time to
repay long-term debt. Borrowing money by issuing a bond is referred to as floating a bond. Sinking is
its opposite, repaying debt or acquiring capital assets without debt. Hence option a is the correct
answer.
• This fund is set aside to periodically redeem bonds, debentures, and preferred stocks. The fund is
accumulated from earnings, and payments into the fund may be based on either a fixed percentage of the
outstanding debt or a fixed percentage of profits.

Q 98.D
• Public expenditure may be classified into developmental and non-developmental expenditures. Former
includes the expenditure incurred on social and community services, economic services, etc. non-
developmental expenditure is a necessary evil.
• Non-developmental expenditure includes
o expenditures made for administrative service, defence service,
▪ debt servicing,
▪ subsidies, etc.
o Cost of tax collection,
▪ cost of an audit,
▪ printing of notes,
o internal law and order,
▪ Pension to retired govt. employees
▪ non-developmental assistance to states is also included in this category.
• Hence option (d) is the correct answer.

Q 99.B
• The yield of a bond is the effective rate of return that it earns. But the rate of return is not fixed — it
changes with the price of the bond. A yield curve is a graphical representation of yields for bonds over
different time horizons. Typically, the term is used for government bonds — which come with the same
sovereign guarantee.
• In a healthy economy, bondholders typically demand to be paid more — or receive a higher “yield” — on
longer-term bonds than they do for short-term bonds. That’s because longer term bonds require people to
lock their money up for a greater period of time — and investors want to be compensated for that risk. In
contrast, bonds that require investors to make shorter time commitments say for three months, don’t
require as much sacrifice and usually pay less.

33 www.visionias.in ©Vision IAS


• Statement 1 is not correct: An inverted Bond yield curve happens when the yield on a longer tenure
bond becomes less than the yield for a shorter tenure bond.

• Statement 2 is correct: A yield inversion typically portends a recession. It shows that investors expect
the future growth to fall sharply; in other words, the demand for money would be much lower than what it
is today and hence the yields are also lower.
• The Bond yield curve has inverted before every U.S. recession since 1955, although it sometimes happens
months or years before the recession starts. Because of that link, substantial and long-lasting inversions of
the yield curve are largely viewed as a strong predictor that a downturn is on the way.

Q 100.B
• The economic capital framework provides a methodology for determining the appropriate level of risk
provisions and profit distribution to be made under Section 47 of the RBI Act, 1934. As per this
provision, the central bank is required to pay balance of its profits to the central government after making
provision for bad and doubtful debts, depreciation in assets, and contributions to staff.
• Last year, RBI formed a committee under the chairmanship of Bimal Jalan to review the provisions
under the Economic Capital Framework. Recently, based on the recommendations of the committee,
the RBI Central Board has decided to increase its net transfer to the government. The recent transfer
includes Rs 1.23 lakh crore of surplus for 2018-19 and Rs 52,637 crore of excess provisions identified
under a revised Economic Capital Framework (ECF) adopted by the RBI board.

You might also like